LSAT and Law School Admissions Forum

Get expert LSAT preparation and law school admissions advice from PowerScore Test Preparation.

 Jeremy Press
PowerScore Staff
  • PowerScore Staff
  • Posts: 1000
  • Joined: Jun 12, 2017
|
#100754
Complete Question Explanation

Weaken. The correct answer choice is (A).

The consumer advocate begins the argument by observing the possibility of harm that can come from drug-related interactions, using the example of aspirin's interaction with fruit juice and the harm it can cause of unnecessary discomfort or taking too much aspirin. Based on the observation and example, the advocate makes a rather sweeping conclusion that the government should require drug companies to disclose to consumers every known drug-related interaction.

While the advocate's argument envisions a potential benefit of disclosing drug interactions (making consumers aware of problematic interactions), it does not tell us whether there might be downsides of such a requirement. This is a major problem for the argument.

The question stem asks for an answer that most weakens the advocate's argument. Focus on the problem we've identified in the argument: let's look for an answer that brings up any downsides of requiring the government to disclose every known drug interaction to consumers. In other words, look for an answer that could suggest the opposite of the conclusion, i.e. that could show that maybe the government should not require the disclosure of every drug interaction.

Answer choice (A): This is the correct answer choice. Answer choice A raises a downside of the proposed conclusion, by suggesting that it might cause consumers to forget about or ignore serious drug interactions. This would suggest that the government should perhaps not go as far as the conclusion wants to go, i.e. that the government should not necessarily disclose every known drug interaction.

Answer choice (B): Answer choice B has no impact on the conclusion, because it does not give any guidance as to whether the interactions it mentions should or should not be disclosed to consumers.

Answer choice (C): Since answer choice C does not provide a significant downside to the disclosures (price increases would be negligible), it does not provide a reason the government should not make the recommended disclosures. Thus, it does not weaken the conclusion.

Answer choice (D): Answer choice D is irrelevant, because the conclusion is limited to the question of whether to disclose the drug interactions we know about (notice the conclusion's language, requiring notification of "all known drug-related interactions").

Answer choice (E): Answer choice E is irrelevant, because what pharmacists usually do does not tell us whether there could be some benefit from doing something different or more than what they usually do. Thus, answer choice E does not weaken the conclusion.
User avatar
 Trying My Best
  • Posts: 6
  • Joined: Jan 05, 2022
|
#93199
I had it narrowed down between letter A and letter C. I am especially frustrated that I got this question wrong because it has a 77% accuracy and letter C was not the most popular incorrect answer choice. What drew me to letter C was that it says that the author's conclusion (requiring drug companies to notify consumers of all known drug-related interactions) would result in ONLY negligible price increases for consumers. I thought, if this was the ONLY result from the author's claim, it would not do what the author is advocating for: reducing harm to patients. Can someone explain how my logic on letter C is incorrect?
User avatar
 DaishiMT
  • Posts: 4
  • Joined: Jan 10, 2022
|
#93232
Bumping for a review for this question. Why is it A? Thanks!
 Robert Carroll
PowerScore Staff
  • PowerScore Staff
  • Posts: 1787
  • Joined: Dec 06, 2013
|
#93239
Trying My Best,

Saying that providing the information would "result in only negligible price increases" does not mean that the only result would be negligible price increases. Compare:

"Taking Route 79 would result only in a slight delay."

vs

"Taking Route 79 would result in only a slight delay."

The first is saying that the only effect would be a slight delay; the second is saying that an effect would be a slight delay, with "only" modifying "a slight" to indicate that that delay wouldn't be much of a big deal. The second example does not attempt to list all the effects of taking Route 79, but just one of them. The first example is, indeed, saying that it's listing all the effects.

You misread answer choice (C) as like the first example. Answer choice (C) is not saying that the only effect will be a price increase, but that a negligible (and only negligible, not any worse) price increase would be the result. Because answer choice (C) does not limit what other good or bad effects might happen, it's not relevant.

Trying My Best and DaishiMT,

If consumers were notified of all drug-related interactions, they'd be notified of all the minor ones, if any. Answer choice (A) gives us a reason for thinking this could actually harm consumers, so it's not strictly a benefit, which weakens the argument.

Robert Carroll
User avatar
 Shonan
  • Posts: 5
  • Joined: Jan 07, 2021
|
#93332
Is answer E wrong because it uses the ambiguous word "printed warnings"?What if the answer says"pharmacies always provide drugs with printed warnings about all known drug interactions"? If that's the case, then the conclusion "government should require drug companies to notify" seems to be weakened.
 Robert Carroll
PowerScore Staff
  • PowerScore Staff
  • Posts: 1787
  • Joined: Dec 06, 2013
|
#93415
Shonan,

I don't think answer choice (E) weakens the conclusion at all - if the government doesn't require notification, as the conclusion contemplates, would there even be printed warnings? The printed warnings themselves could be an effect of government mandates. Thus, it's not clear that answer choice (E) is anything other than consistent with the argument.

Robert Carroll
 frk215
  • Posts: 33
  • Joined: Sep 07, 2020
|
#95576
Hello! I didn't really like any of the answers below this question, but I found myself stuck between A and E.

My logic for E was the following: with a weaken question, we would assume the answer choices are correct/true and approach the stimulus with suspicion. E states "Pharmacists usually draw patients' attention to printed warnings that are provided with drugs." so we can assume that there are printed warnings, that are pointed out to pharmacists.

The stimulus suggests that the problem (patients being unaware of minor drug interactions) can be solved/should be responded to with a government mandate for drug companies to notify patients of all known drug interactions.

I guess I justified E by assuming that the printed warnings do already include all those minor drug related interactions. Buttt that's a major leap, that's entirely unjustified by the stimulus (just no support there). So would E be wrong because even if it is true, we
1. don't know that the warnings contain the information that author wants pointed out
2. "usually" as stated in e isn't strong enough; the author in concluding the need for a requirement is advocating for all known drug interactions to be reported by all companies all of the time
3. because of the things stated above ^ it doesn't or really even affect the argument/the relationship between the premise and the conclusion

Soooo. It's A. Is there anything missing from my analysis or an inference that I took too far? Thanks for the help team! :)
 Adam Tyson
PowerScore Staff
  • PowerScore Staff
  • Posts: 5271
  • Joined: Apr 14, 2011
|
#96303
I think your analysis is pretty good, frk215, and shows why E is not a good answer choice. Also consider that pharmacists pointing out printed warnings does not mean that the minor interactions are already in those warnings. The author might be arguing that they should be added to the printed warnings so that the pharmacists could point them out.

Focus on the conclusion: the gov't should do this. Now, think about how to weaken that. What do we need the answer to do? We need it to suggest that doing this is a bad idea. Not just that we don't need to do it, but that we should maybe NOT do it. Look for something that makes this proposal a bad idea, where doing what the author suggested has harms that could outweigh the benefits. Don't just pick something passive like "this may not be totally necessary." Look to attack the claim that we should do it by giving evidence that we shouldn't! Only answer A gives us that level of harmful results that would make this look like a bad idea.

Get the most out of your LSAT Prep Plus subscription.

Analyze and track your performance with our Testing and Analytics Package.